Inscription / Connexion Nouveau Sujet
Niveau Reprise d'études-Ter
Partager :

Fermat n=3

Posté par
fabo34
04-08-22 à 09:05

Bonjour,

  J'ai trouvé le papier d'un chinois qui propose une nouvelle démonstration de impossibilité de x3 + y3= z3, soit disant compréhensible par un élève de lycée.

C'est ici: https://www.mathematicaljournal.com/article/9/1-1-17-231.pdf

J'ai besoin de nos lumières, car je n'y comprends rien.

Posté par
ty59847
re : Fermat n=3 04-08-22 à 11:21

On trouve souvent des papiers qui disent : Je suis très brillant, j'ai découvert la solution de tel problème qui résistait à tous les mathématiciens, voici cette solution, et comme vous le voyez, cette solution est ultra-accessible.
Et on voit derrière un truc, qui ressemble à des maths, mais qui est truffé d'imprécisions et d'erreurs.
Laisse tomber.

Posté par
Ulmiere
re : Fermat n=3 04-08-22 à 11:59

D'où vient le type qui a écrit ça ?

Citation :
Department of Physics, Hunan
Normal University, Chang
Sha, China, 410083


Ah ben oui, ceci explique cela
L'auteur a l'air de penser qu'une solution de l'équation de Fermat s'écrit forcément {x,y,z} = {1+t, a, b} avec {t,a,b} solution de la même équiation de Fermat

Posté par
fabo34
re : Fermat n=3 04-08-22 à 12:22

Mince! Moi qui me faisais une joie de voir une nouvelle démonstration!

C'est fou. Apparemment, ce journal Journal of Mathematical Problems, Equations and Statistics se dit à "evaluation par les pairs" ( Journal of Mathematical Problems, Equations and Statistics is a Peer Reviewed International Open Access Journal which will be abstracted in various reputed databases. The Journal provides the platform with the aim of motivating the students and personnel. )

Je n'y connais rien en terme de publication scientifique. L'article est dit reçu le 07/01/2020, et accepté le 10/03/2020.

Ça arrive souvent, ce genre de bévues ?

Posté par
Ulmiere
re : Fermat n=3 04-08-22 à 12:35

Je ne puis juger de ce que je ne sais. Peut-être que l'auteur essaie de le montrer dans un cas bien particulier issu de la physique (peut être que les x,y,z sont des spins entiers, qui sait), d'où les translations dans tous les sens. Ce expliquerait les t_{min}, t_{max} et compagnie qui sortent de nulle part.

Ce qui est sûr c'est que ce n'est pas une manière correcte de présenter ses résultats pour un mathématicien. Quand on prétend s'attaquer à un classique comme le dernier théorème de Fermat pour le vulgariser, on doit au moins énoncer le théorème qu'on essaie de démontrer. La syntaxe est également douteuse, l'auteur utilise des mots réservés aux amateurs

Citation :
it proved that there are no a group
of positive integers of x, y, and z


Par exemple, ça, ça craint vraiment. Le mot groupe en mathématiques, désigne quelque chose de bien particulier, pas simplement un ensemble.


Il est possible de démontrer le cas n = 3 avec des outils de base, mais personnellement  je ne connais pas de preuve qui ne demande un minimum d'explications à certains points précis.

Tu peux regarder dans les références de l'article du chinois qui tu as joint, d'autres chercheurs ont sans doute déjà fait le travail correctement

Posté par
ty59847
re : Fermat n=3 04-08-22 à 12:59

C'est sur les questions d'arithmétique essentiellement qu'il y a le plus souvent ces problèmes de publications erronées.

En arithmétique, l'énoncé du problème est souvent simple : l'équation a3+b3=c3 a-t-elle des solutions entières.   On ne peut pas faire plus simple. N'importe quel lycéen peut prendre un papier et un crayon, et se lancer dans des recherches/élucubrations sur ce thème.
Et pourtant, même si l'énoncé paraît simple, la résolution est hyper-compliquée, voire inconnue.

En plus, sur des questions comme celle-ci : telle équation a-t-elle des solutions entières, l'avènement de l'informatique fait que  n'importe quel lycéen peut faire plein de tests. On fait un programme qui teste tous les entiers un par un ... enfin, presque tous les entiers, euhhhh,  on teste les entiers entre 1 et 100000000000, et on regarde.

En géométrie, en algèbre, on a beaucoup moins d'apprenti sorciers qui croient avoir solutionné tel problème supposé insoluble.

Posté par
fabo34
re : Fermat n=3 04-08-22 à 13:36

Merci pour vos précisions.

Je me permets de vous relancer : qui sont les "pairs" qui aurait accepté ce travail? Dans un journal à revue, sont-ils cités
dans l'article (histoire qu'on sache qui valide? S'il n'y a pas de copinage, ...)

Posté par
fabo34
re : Fermat n=3 04-08-22 à 14:02

Concernant les autres auteurs cités, il y a un James Joseph, un mathématicien retraité de l'univ de Washington, qui a carrément publié une autre preuve ∀n, en la comparant à celle de Wiles (qu'il semble donc avoir comprise!): ici https://www.researchgate.net/profile/James-Joseph-19/publication/291418125_A_proof_of_Fermat%27s_last_Theorem_using_elementary_algebra/links/56cfb3ad08ae4d8d649fc9b7/A-proof-of-Fermats-last-Theorem-using-elementary-algebra.pdf

Celle là aussi serait fausse? (Ça ressemble à ma tentative désespérée via les polynômes de Girard !!! polynômes de Newton-Girard et xⁿ+yⁿ+zⁿ=0ⁿ )

Posté par
ty59847
re : Fermat n=3 04-08-22 à 15:27

Un truc qui porte comme titre 'Preuve du dernier théorème de Fermat utilisant l'algèbre élémentaire', ça pue l'arnaque à plein nez.
Un matheux respectable ne va pas chercher à humilier des générations de matheux, en disant qu'il existe une preuve qui ne nécessiterait que des notions élémentaires d'algèbre.
Un lien comme ça, je ne l'ouvre même pas, à cause du mot 'élémentaire'.

Posté par
fabo34
re : Fermat n=3 04-08-22 à 16:18

ty59847 : c'est dommage. Au moins pour me dire où est l'erreur. Moi je crois voir que tout ce qu'il écrit est vrai dans le cas n \nmid xyz. Après, s'il arrive à un résultat plus général que celui de Sophie Germain, ce serait pas mal!  Il écrit des trucs modulo n² ... Là  je suis perdu

Ici encore, c'est reçu Le 02/02/2015, et accepté le 31/03/2015. Comment est-ce possible?

Et puis, pour lui "élémentaire" signifie "with techniques within de Graps of Fermat" . Ce n'est donc pas forcément pour humilier les autres. Personnellement, je crois au génie, donc à la possibilité qu'un homme voit un jour ce que les autres n'ont jamais su voir. Encore ces jours-ci, il a une émission de France-Culture  sur Champollion : beaucoup de ces contemporains avait pourtant tout pour déchiffrer les hiéroglyphes, seul lui l'a fait. Idem Einstein. Et bien d'autres exemples. C'est humiliant, mais faut l'accepter

Alors, il vaut quoi, ce papier de James Joseph ?

Posté par
Sylvieg Moderateur
re : Fermat n=3 04-08-22 à 18:54

Bonjour,
Je me permets une mise au point :
L'article dont vous parlez ne s'intéresse qu'à un cas particulier du théorème de Fermat, le cas n = 3.
En cherchant "théorème de Fermat n = 3" dans votre moteur de recherche préféré, vous trouverez des démonstration plus convaincantes.
Pourquoi se polariser sur celle-ci ?

Posté par
Ulmiere
re : Fermat n=3 04-08-22 à 19:29

fabo34 @ 04-08-2022 à 16:18

ty59847 : c'est dommage. Au moins pour me dire où est l'erreur. Moi je crois voir que tout ce qu'il écrit est vrai dans le cas n \nmid xyz. Après, s'il arrive à un résultat plus général que celui de Sophie Germain, ce serait pas mal!  Il écrit des trucs modulo n² ... Là  je suis perdu

Ici encore, c'est reçu Le 02/02/2015, et accepté le 31/03/2015. Comment est-ce possible?

Et puis, pour lui "élémentaire" signifie "with techniques within de Graps of Fermat" . Ce n'est donc pas forcément pour humilier les autres. Personnellement, je crois au génie, donc à la possibilité qu'un homme voit un jour ce que les autres n'ont jamais su voir. Encore ces jours-ci, il a une émission de France-Culture  sur Champollion : beaucoup de ces contemporains avait pourtant tout pour déchiffrer les hiéroglyphes, seul lui l'a fait. Idem Einstein. Et bien d'autres exemples. C'est humiliant, mais faut l'accepter

Alors, il vaut quoi, ce papier de James Joseph ?


Loin de moi l'idée de contester le génie d'Einstein, mais beaucoup des ses contemporains auraient pu être à sa place. Poincaré et Lorentz par exemple avaient déjà fait l'étude de base du groupe de Lorentz qui est la première chose qu'on enseigne aux étudiants qui apprennent la relativité (restreinte). Ce pauvre Poincaré avait aussi mis le doit sur les bases de la physique quantique avant que Planck ne lui vole la vedette, vraiment pas de bol
C'était sûr que tôt ou tard, quelqu'un allait mettre le doigt sur la notion de variété riemannienne et Einstein l'a fait, mais pas tout seul. Le tenseur d'Einstein s'écrit en utilisant celui de Christoffel .

Sinon pour en revenir au sujet, des preuves dites élémentaires existent pour n = 3, mais il faut au moins connaitre les entiers de Gauss pour les comprendre.
Galois est un génie qui est mort à 19 ans et tous les algébristes savent ce que les mathématiques lui doivent. Un jour on aura peut-être une nouvelle branche des mathématiques qui trivialise l'arithmétique modulaire, mais d'ici là y'a peu de chances de pondre une preuve de la conjecture de Syracuse qui tienne sur moins de 200 pages bien tassées

Posté par
fabo34
re : Fermat n=3 04-08-22 à 20:02

Sylvieg: bonjour. Je cite 2 articles. Le premier est effectivement celui de Zhang Yue pour le cas n=3. L'autre, de James Joseph, qui y est référencé, concerne tous les n premiers.

Je ne me focalise pas. Ce sont juste de nouvelles publications (2020 et 2015) qui apparaissent en premier dans les moteurs.

Oui, le cas n=3 est riche. Moi même j'avais essayé une méthode en utilisant la technique des triplets pythagoriciens en passant dans ℚ( faisable par un lycéen de 1ère). Ça débouchait sur une nouvelle impossibilité diophantienne que j'avais postée ici Equation diophantienne. Apparemment elle n'apporte rien ou n'est pas résoluble; en tout cas elle n'a pas inspiré foule. Personnellement je bute.

Quand quelqu'un publie (dans un journal à revue!) une nouvelle technique, on ne peut qu'être enthousiaste, non? À moin niveau, comment savoir si c'est bidon!!?

Concernant le cas James Joseph, sauriez-vous me dire où est la fausseté de sa démonstration  ? Où dois-je ouvrir un autre fil pour ça?

Posté par
Ulmiere
re : Fermat n=3 04-08-22 à 21:11

J'ai pas regardé dans le détail, mais dans le second article, il ne s'intéresse qu'au cas où n est un nombre premier impair et x,y,z sont copremiers deux à deux. C'est plus restrictif que le théorème de Fermat.
Le fait qu'on puisse se ramener à presque ce cas là est lui même un théorème qu'il convient de démontrer.

Le début du premier lemme est trivial en utilisant le morphisme de Fröbenius. L'auteur opte pour une démonstration directe à base de coefficents binomiaux. C'est aussi comme ça qu'on montre que f est un morphisme d'anneaux de toute façon. Y'a quelques subtilités.

Ensuite il montre que les solutions éventuelles du problème sont multiples de l'exposant via un calcul que je n'ai lu qu'en diagonale. Mais dans ce cas, ils ont un facteur commun > 1, et donc ne sont pas premiers entre eux. Contradiction donc pas de solution.

Faudrait que je lise avec attention pour te dire si je trouve une erreur, mais là j'ai pas trop le temps

Posté par
fabo34
re : Fermat n=3 05-08-22 à 10:00

Ulmiere : Merci. J'ai ouvert un nouveau fil ici : Fermat-Wiles: "nouvelle" preuve de 2015? . C'est les vacances pour certains, avec peut-être plus de temps

Posté par
fabo34
re : Fermat n=3 05-08-22 à 18:57

Bon. Tous ces articles sont bidons.
Désolé. Je me méfierai la prochaine fois avant de télécharger une publication sur internet.

Je vais demander au modérateur d'effacer ce fil

Encore merci
Et désolé

Posté par
malou Webmaster
re : Fermat n=3 05-08-22 à 19:08

Bonsoir
Nous n'effaçons pas les échanges sur notre site.



Vous devez être membre accéder à ce service...

Pas encore inscrit ?

1 compte par personne, multi-compte interdit !

Ou identifiez-vous :


Rester sur la page

Inscription gratuite

Fiches en rapport

parmi 1675 fiches de maths

Désolé, votre version d'Internet Explorer est plus que périmée ! Merci de le mettre à jour ou de télécharger Firefox ou Google Chrome pour utiliser le site. Votre ordinateur vous remerciera !